Property Flashcards
Under the Rule of Convenience, …
…a class closes when a member of the class is entitled to distribution.
If the class is closed when the interest takes effect (i.e. when the testator died), then all members of the class alive at that time will take, and any members of the class later born or conceived will not take.
T devised to grandchildren who reach the age of 25. A is 27 at the time of the testator’s death and is entitled to immediate distribution. Therefore, the class closes immediately. All grandchildren born when the class closed (A, B, C, and D) will be allowed to take when they reach 25. No grandchildren born after the testator’s death will take.
Where one tenant makes improvements to the property that may increase the value of the property but are not “necessary,” the other tenant is not liable unless there was an agreement between the parties to contribute to the cost. Thus, a tenant who makes improvements to property cannot seek immediate reimbursement from a co-tenant in a suit for contribution where there was no agreement between the parties to contribute to the cost.
BUT
Can tenant file partition action to recover the increase in value of the property?
However, a tenant who makes improvements to a property may file a partition action and request an accounting. If the court grants a partition by sale, then the property will be sold, and the proceeds will be distributed to the tenants. The remaining value will be divided between the tenants according to their shares.
Rule Against Perpetuities - what is it?
No interest in property is valid unless it must vest, if at all, not later than 21 years after some
life in being (“measuring life”) at the creation of the interest.
If there is any possibility that the
interest might vest more than 21 years after a life in being, the interest is void.
RAP - when does it apply?
The Rule applies to
contingent remainders,
executory interests,
vested remainders subject to open (class gifts),
options to purchase (not attached to a leasehold),
rights of first refusal, and
powers of appointment.
When Perpetuities Period Begins to Run?
For interests granted by will, it runs from the date of the
testator’s death;
for deeds, it is the date of delivery.
The period runs on an irrevocable trust
from the date it is created;
it runs on a revocable trust from the date it becomes irrevocable.
An interest vests for purposes of the Rule when it becomes:
(i) possessory, or
(ii) an indefeasibly
vested remainder or a vested remainder subject to total divestment.
Interests Exempt from Rule
Except for vested remainders subject to open, the Rule Against Perpetuities does not apply to vested interests. Thus, other vested remainders, reversions, possibilities of reverter, and rights
of entry are not subject to the Rule.
Moreover, there is a charity-to-charity exception to the Rule (i.e., the Rule does not apply to any disposition over from one charity to another), and an exception for options to purchase held by a current tenant
RAP in Operation—Common Pitfall Cases
Executory Interest Following Defeasible Fee
Generally, an executory interest that follows a defeasible fee (e.g., “to A for so long as no liquor is consumed on the premises, then to B”) violates the Rule Against Perpetuities, and the executory interest is stricken. (An executory interest following a defeasible fee is valid only if the condition is specific to the fee holder or expressly limited to the perpetuities period.)
RAP in Operation—Common Pitfall Cases
Age Contingency Beyond Age Twenty-One in Open Class
A gift to an open class conditioned on members surviving beyond age 21 violates the Rule.
Example: “To A for life, then to those of A’s children who attain the age of 25.”
The remainder in A’s children violates the Rule and is void.
Some states have enacted perpetuities reform legislation that reduces such age contingencies
to 21.
RAP in Operation—Common Pitfall Cases
Fertile Octogenarian
A woman is conclusively presumed to be capable of bearing children, regardless of her age or medical condition.
Example: “To A for life, then to A’s children for life, then to A’s grandchildren in fee.” The remainder in A’s grandchildren is invalid despite the fact that A is 80 years old.
A few states have enacted perpetuities reform statutes that raise a presumption that women over a certain age (e.g., 55) cannot bear children. Also, medical testimony regarding a woman’s childbearing capacity is admissible in these states.
RAP in Operation—Common Pitfall Cases
Unborn Widow or Widower
Because a person’s widow (or widower) is not determined until his death, it may turn out to be someone who was not in being at the time of the disposition.
Example: O conveys “to A for life, then to A’s widow for life, then to A’s surviving issue in fee.” In the absence of a statute to the contrary, the gift to A’s issue is invalid because A’s widow might be a spouse who was not in being when the interest was created.
Compare: A remainder “to A’s children” would be valid because, unlike issue, they would be determined at A’s death.
Where necessary to sustain a gift, a few state statutes raise a presumption that any reference to a person’s spouse, widow, or widower is to a person in being at the time of the transfer.
RAP in Operation—Common Pitfall Cases
Administrative Contingency
A gift conditioned on an administrative contingency (e.g., admission of will to probate) violates the Rule.
Example: A gift “to my issue surviving at the distribution of my estate” is invalid because the estate might be administered beyond the period of the Rule.
A few state reform statutes eliminate this problem by raising a presumption that the transferor intended that the contingency should occur, if at all, within 21 years.
RAP in Operation—Common Pitfall Cases
Options and Rights of First Refusal
1) Options
An option is a contract, supported by consideration, that creates in the optioned a right to purchase the property on terms provided in the option. Generally, an option to purchase that is structured so that it might be exercised later than the end of the perpetuities period is void. (A significant minority of courts will construe the option as lasting only for a reasonable time.)
Exception: The Rule Against Perpetuities does not apply to options to purchase held by the current lessee.
Watch for a fact pattern on the exam where a tenant has an option to purchase beyond the perpetuities period. Remember that the Rule does not apply to such an option held by a current tenant or his assignee, but it does apply to
a former tenant and to any party to whom the current tenant might transfer the option separately from the lease (in jurisdictions permitting such a transfer).
2) Rights of First Refusal
In contrast, rights of first refusal in many states are presumed to be personal to the holder and therefore not subject to the Rule Against Perpetuities. In some states, however, rights of first refusal are subject to the Rule in the same manner as
options.
Application of the RAP to Class Gifts
**_“Bad-as-to-One, Bad-as-to-All” Rule_** If the interest of any class member may vest too remotely, the whole class gift fails. For the class gift to vest, the class must be closed and all conditions precedent must be satisfied for every member.
**“Gift to Subclass” Exception** Each gift to a subclass may be treated as a separate gift under the Rule.
Example: “Income to A for life, then to A’s children for their lives. Upon the death of each of A’s children, the corpus is to be distributed to that child’s issue, per stirpes.” The gifts to each of A’s children’s issue are considered separately. Thus, the gifts to issue of A’s children living at the time of the
disposition are good, but the gifts to the issue of afterborn children of A violate the Rule and are void.
**Per Capita Gift Exception** A gift of a fixed amount to each member of a class is not treated as a class gift under the Rule.
Example: “$1,000 to each of my great-grandchildren, whether born before or after my death.” This creates gifts to individuals, each of whom is judged separately under the Rule.
A possibility of reverter is devisable. Example?
After the land owner died, leaving her entire estate to her daughter, the daughter is the holder of the possibility of reverter.
To my son for life, reminder to my son’s children and their heirs. At the time of T’s death, the son had one child. Two years after T’s death, the son had another child.
When the testator dies, the son has a present possessory life estate and the son’s first child, who is ascertainable because he/she was born before testator’s death, has a vested remainder subject to open because the remainder was devised to a class: the son’s children.
When the second child is born, the second child is added to the class and also has a vested remainder subject to open because the son could have more children that join the class.
Under the Doctrine of Worthier Title…
…. the land owner retains a reversion in the property. Upon A’s death, the property reverts to the land owner in fee simple absolute.
It applies when A receives an estate less than a fee simple (i.e. a life estate); the land owner’s heirs receive a remainder; both interests created by the same instrument; and both interests are legal.
A restraint on alienation is a condition on the ownership of real estate that restricts conveyance. A forfeiture restraint provides that the grantee loses the estate if the grantee attempts to convey the property. A forfeiture restraint is permitted for…
…life estates and future interests but is not enforceable for fee simple estates. The grant to the son–“to my son and his heirs”–is a fee simple estate.
When joint tenants mortgage the property together, the mortgage….
…does not sever the joint tenancy.
In addition, a joint tenant may sever the joint tenancy by voluntarily or involuntarily transferring her interest in the property, but a joint tenant cannot devise her interest.
Zoning violation v. zoning restriction
While a zoning violation may make title unmarketable, a zoning restriction on the property does not make title unmarketable.
Under the Uniform Vendor and Purchaser Risk Act, absent a written agreement to the contrary, the risk of loss is placed on …
…seller during the time between execution of a binding contract and the closing date.
However, that risk transfers to the purchaser if legal title or possession has passed to the purchaser.
When a buyer breaches a contract to sell real property, the seller can recover …. consequential damages, such as mortgage interest payments that the seller pays after the buyer’s breach.
foreseeable
The seller cannot recover unforeseeable consequential damages.
When a buyer breaches a contract to sell real property, the seller can recover expectation damages. Expectation damages are measured by the difference between the contract price and the market price of the real property at the time of the breach.
When a buyer willfully breaches a contract to sell real property, the seller can recover punitive damages from the buyer.
When a buyer breaches a contract to sell real property, the seller can recover reasonable reliance damages. Reasonable reliance damages include the costs of inspections paid by the seller.
The right of the mortgagor to pay off the mortgage debt after default but before foreclosure (known as the right of redemption) … be waived at the time the mortgage is created.
may not
Redemption is permitted at any time after default and before the foreclosure sale.
Many states permit redemption even after the foreclosure sale, if the redemption occurs within the statutorily designated time (usually six to 12 months after the foreclosure sale).
The proceeds of a foreclosure sale go first to reimburse….
…the costs of the foreclosure sale.
Assumption
If the grantee signs an assumption agreement, he becomes primarily liable to the lender, while the original mortgagor is secondarily liable as a surety.
However, the mortgagee may opt to sue either the grantee or the original mortgagor on the debt.
If no assumption agreement is signed, the grantee is not personally liable on the loan, and the original mortgagor remains primarily and personally liable.
Nonetheless, if the grantee does not pay, the loan may be foreclosed, wiping out the grantee’s investment.
Priorities
A mortgage’s priority is usually determined by the time it was placed on the property.
Foreclosure does not destroy any interests senior to the interest being foreclosed.
It generally destroys all junior interests, but failure to include a junior interest holder in a foreclosure
action results in preservation of that party’s interest.
Modification of Priority
Although priority among mortgages is generally determined by chronology, this priority may be changed by:
(i) the operation of the recording statute if a prior mortgagee fails to record;
- *(ii) a subordination agreement between a senior and junior mortgagee;**
- *(iii) a purchase money mortgage;**
(iv) the modification of a senior mortgage (junior
mortgage has priority over the modification);
- *(v) the granting of optional future advances by a mortgagee with notice of a junior lien (junior lien has priority over
advances) ; or**
(vi) subrogation (a senior mortgage is refinanced with a new mortgage)
A purchase money mortgage (“PMM”) is …
…a mortgage given in exchange for funds used to purchase the property.
PMMs are given either to the seller as part of the purchase price or to a third-party lender.
PMMs have priority over mortgages, liens, and other
claims against the mortgagor that arise prior to the mortgagor’s acquisition of title.
However, subsequent mortgages or liens may defeat PMM priority by operation of the recording acts.
As between two PMMs, a seller’s mortgage generally has priority over a third party’s. If there are two third-party PMMs, priority is determined by chronological order.
Proceeds of Foreclosure Sale
Proceeds are applied first to the expenses of the sale, attorneys’ fees, and court costs;
then to
the principal and accrued interest on the foreclosed loan;
next to any other junior interests
in the order of their priority;
and finally to the mortgagor.
A person with a remainder interest in mortgaged property has a duty to pay the …, and the life tenant has a duty to pay the … on the mortgage.
principal
interest
“subject to the mortgage”
v.
“assumes the mortgage”
If a mortgagor transfers the property “subject to the mortgage” and mortgage payments are not made, the mortgagee may foreclose and force the property to be sold, but the transferee does not have personal liability for the debt.
In contrast, if the transferee of mortgaged real property “assumes the mortgage” and mortgage payments are not made, the mortgagee may foreclose and force the property to be sold, and, in addition, the transferee is personally liable for any deficiency.
Finally, in a novation, the transferee of real property and the mortgagee agree that the transferee will assume the mortgage and the mortgagor will be released from liability.
Emblements (or fructus industriales) are ….
…annual growing crops resulting from human cultivation.
Emblements are considered part of the land and pass with a conveyance of the land unless the grantor expressly reserves the crop in the deed of conveyance.
i.e.
The farmer conveyed the land to the buyer and did not expressly reserve the pumpkin crop. Therefore, the pumpkins passed to the buyer with the conveyance of the land.
An easement by estoppel.
An easement by estoppel may be created if an act or representation of the owner of the servient estate causes justifiable reliance by the owner of the dominant estate, and the owner of the benefited estate would suffer damages if the easement is not recognized.
An easement by prescription.
An easement by prescription is the easement equivalent of adverse possession. To create an affirmative easement by prescription, the use of the property must be open and notorious, actual, continuous for the prescription period, hostile, and exclusive.
An easement by necessity.
An easement by necessity is created where there is severance of title to land held in common ownership and strict necessity for the easement at the time of severance.
An easement implied by prior use.
An easement implied by prior use is created where there is a severance of title to land held in common ownership;
an existing, apparent, and continuous use when severance occurs;
and the parties intended use to continue after division of land.
license - transferable?
A license is personal to the licensee, and therefore it is not transferable unless the licensor intended that the license be transferable.
The language in the lease created a restriction on the use of the property. The question is whether the restriction runs with the land to the sublessee.
The restriction certainly touches and concerns the land, it can therefore be presumed that it was intended that the restriction run with the land, and the sublessee took with notice of the restriction because the lease was recorded. The problem is with vertical privity between the lessee and the sublessee. On the servient side vertical privity exists only if there is a transfer of the entire interest in the land. The lessee transferred only five years of the 45 years remaining on the lease, so vertical privity is lacking. As a result, the restriction could not be enforced as a covenant, which means the lessor could not recover damages.
However, since privity is not an element of an equitable servitude, the lessor could enforce the restriction as an equitable servitude and get an injunction to prevent the violation of the restriction.
At common law, if the lease was silent, the …. had a duty to repair.
tenant
Under the doctrine of constructive adverse possession,….
…an adverse possessor who enters under “color of title” (i.e., with a facially valid but for some other reason invalid deed) will be deemed in constructive possession of all the property described in the deed so long as the portion of the property constructively possessed is reasonable in proportion to the property actually possessed.
Notice statute
No conveyance, transfer, or mortgage of real property shall be good against subsequent purchasers for value and without notice, unless the same be recorded.
“No conveyance or mortgage of an
interest in land is valid against any
subsequent purchaser for value
without notice thereof, unless it is
recorded.”
To prevail under a notice statute, a person must: take subsequent in time to another person claiming ownership, be a bona fide purchaser for value, and take the property without notice.
(Last BFP)
Subsequent bona fide purchaser
(i.e., for value, without notice)
prevails.
Race-notice statute
Every conveyance of real property is void as against a subsequent purchaser in good faith and for value whose conveyance is first duly recorded, unless the conveyance shall have been duly recorded prior to the record notice of action.
“No conveyance or mortgage of an
interest in land is valid against any
subsequent purchaser for value
without notice thereof whose
conveyance is first recorded.”
Under a race-notice statute, to prevail a claimant must: take subsequent to another person claiming ownership, be a bona fide purchaser for value, take without notice, and record first. Notice can be actual, constructive, or inquiry notice.
(BFP who records 1st)
Subsequent bona fide purchaser
(i.e., for value, without notice) who
records first prevails.
Race Statutes
“No conveyance or mortgage of an
interest in land is valid against any
subsequent purchaser whose
conveyance is first recorded.”
Under a pure race statute, whoever records first wins.
Notice is irrelevant.
Very few
states have such statutes.
Grantee who records first prevails.
*** If neither buyer records, the common law rule of first in time, first in right applies.
Is an oral contract for the sale of land ever valid?
The sale of land normally needs to be in writing to satisfy the Statute of Frauds.
But, there is an exception for part performance. An oral contract for the sale of land is valid with a showing of a combination of any, or all three, of the following: (1) payment of all or part of the purchase price; (2) taking of possession; and (3) making substantial improvements to the property.
What are valid arguments when seeking to rezone property?
The rezoning request is consistent with the long-range land use plans of the local government.
There was an error in the original zoning classification of the buyer’s property.
Changes have occurred in the buyer’s neighborhood which prevent the use of his land as originally zoned.
NOT: Providing evidence to the rezoning board that the seller misinformed the buyer that his intended use would be permitted. Arguing to the board that the land-sale contract was invalid or misconstrued is a remedy best sought under a breach of contract action, not to justify a rezoning of the property.
If a senior mortgage is modified, a junior mortgage prevails over the modification if …
…the modification materially prejudices the holder of the junior mortgage, such as by:
(1) increasing the amount of principal; or
(2) increasing the interest rate (if the rate under the original mortgage was fixed).
Modifications that normally do not materially prejudice the holder of the junior mortgage include:
(1) extension of the mortgage maturity date; and (
2) rescheduling installment payments.
i. e.
Here, the term extension did not materially prejudice Junior Bank’s interest because it lowered the monthly payment. Senior Bank’s mortgage maintained priority despite the modification.
A developer owned an office building
subject to a first mortgage with a creditor in the
amount of $1 million. Subsequently, the developer
borrowed $100,000 from a bank secured
by a second mortgage on the building to help
pay the first mortgage and other expenses of
the building. The developer’s financial condition
worsened, and he was unable to make the
required payments on the first mortgage to the
creditor. The developer approached the creditor
and offered to give her a deed to the building
in satisfaction of all of his obligations to the
creditor. The developer delivered to the creditor
a quitclaim deed to the building, which recited
as consideration the release of the developer
from all liability on the mortgage to the creditor.
The deed was duly recorded.
Shortly thereafter, the office market greatly
improved, and the building was worth $1.5
million. The developer then brought an action
against the creditor, claiming that the deed was
an equitable mortgage, and the bank served
notice on the creditor that it was preparing to
foreclose its mortgage on the building.
Against which parties will the creditor
prevail?
(A) The developer only.
The conveyance from the developer to the creditor discharged the first mortgage on the property because the conveyance was in satisfaction of the mortgage obligation. The first mortgage, however, was never foreclosed and therefore did not wipe out the second mortgage. When the first mortgage was discharged by the conveyance, the second mortgage became a first mortgage and is still a valid
encumbrance on the property.
The purchaser of a note and trust deed is subrogated to (i.e., takes over) the interest of the seller. What does it mean?
subrogated (i.e., takes over)
The general rule is that when a mortgage is foreclosed,
the buyer at the sale will take title as it existed when the mortgage was placed on the property.
Thus, foreclosure generally will destroy all interests junior to the mortgage being foreclosed, but will not discharge senior interests.
However, those with interests subordinate to those of the foreclosing party are necessary parties to the foreclosure action. Failure to include a necessary party results in the preservation of that party’s interest despite foreclosure and sale.
i.e.
The bank’s original mortgage was senior to the financing company’s mortgage. However, where the landowner enters into a modification agreement with the senior mortgagee, raising its interest rate or otherwise making it more burdensome, the junior mortgagee will be given priority over the modification. Thus, the bank’s modification would not have priority over the financing company’s mortgage. Nevertheless, because the financing company failed to include the bank in its foreclosure action, the bank’s mortgage interest survives under its modified terms, even though the modification did not have priority.
equitable mortgage
A landowner needing to raise money may “sell” the land to a person who pays cash and may give the lender an absolute deed rather than a mortgage.
However, if a court concludes that the deed was really given for security purposes, it will treat it as an equitable mortgage and require that the creditor foreclose it by judicial action, like any other mortgage.
The following factors indicate an equitable mortgage:
(i) the existence of a debt or promise of payment by the deed’s grantor;
(ii) the grantee’s promise to return the land if the debt is paid; (iii) the fact that the amount advanced to the grantor/debtor was much lower than the value of the property;
(iv) the degree of the grantor’s financial distress; and
(v) the parties’ prior negotiations.
At a foreclosure sale, the highest bidder
takes the property. Therefore, to take possession of the property, the friend must not only institute
foreclosure proceedings but also be the successful bidder at the sale.
What is mortgage? What is note?
A mortgage is a security interest in property and a note is evidence of the underlying debt.
Physical possession of the mortgage and
note is not required for ownership.
i.e.
Thus, because the brokerage firm bought the mortgage and
note and recorded its interest, the brokerage firm is the owner of both, even though it left possession
of the documents with the finance company.
The investor has no interest in the mortgage and
note because he had record notice of the brokerage firm’s interest (because the brokerage firm recorded the mortgage). Having notice of the brokerage firm’s interest, the investor cannot claim the protection of the recording act. Also, the investor cannot claim holder in due course status,
because that status requires no notice of any other claims to the property. Therefore, the investor
has no interest in the mortgage and note.
Under the title theory, title is in the mortgagee (lender) until the mortgage has been satisfied or foreclosed.
Thus, the mortgagee is entitled to possession on demand at
any time.
If a junior mortgage is placed on the property and the senior lender later makes an optional advance while having
notice of the junior lien, ….
…the advance will lose priority to the junior lien.
(An optional advance
is one that the senior lender is not contractually bound to make.)
When mortgagor transfer the
mortgaged property, does he
remain personally liable on the mortgage loan?
The mortgagor
remains personally liable on the mortgage loan regardless of any subsequent transfers of the mortgaged property.
When a grantee signs an assumption agreement, promising
to pay the mortgage loan, he becomes primarily liable to the lender (who is a third-party beneficiary of the assumption agreement), while the original mortgagor becomes secondarily liable as a surety.
When a mortgagor sells property and conveys a deed, the grantee takes subject to the mortgage, which remains on the land (unless the proceeds of the sale are used to pay off the mortgage). However, a grantee who does not sign an assumption agreement does not become personally liable on the loan. Instead, the original mortgagor remains primarily and
personally liable.
“due-on-sale” clause
A mortgage containing a “due-on-sale” clause allows the
lender to demand full payment of the loan if the mortgagor transfers an interest in the property without the lender’s consent.
Under federal law, due-on-sale clauses are generally enforceable.
“shelter rule”
In general, a person who takes from a bona fide purchaser will prevail against any interest that the transferor-bona fide purchaser would have prevailed against, even if the person taking the property has actual or record notice of the
prior interest.
Purchase money mortgages
Where a seller of property receives a mortgage as part of the purchase price, a purchase money mortgage results. Purchase money mortgages may also arise when a third party lends money to the buyer for the purchase of property and takes a mortgage on the property in return.
In general, the seller’s purchase money mortgage will take priority over the third-party purchase money mortgage.
Purchase money mortgages, however, are subject to later liens by virtue of recording acts.
In the case where a mortgage is modified by agreement between the parties, any increase in the debt resulting from the modification will be subject to a junior lien, even if the original mortgage itself had priority over the junior lien.
In the same way, an optional (as opposed to an obligatory)
advance that is made after the junior lien will have a lower priority than the junior lien. Again, this is the case even if the original mortgage is first in priority.
A buyer bought a home from a real estate
developer for $700,000. The buyer paid
$100,000 of the purchase price herself. The
buyer’s employer provided $100,000 of the
purchase price by giving the buyer a loan
and taking a mortgage. The developer loaned
$500,000 to the buyer to finance the remainder
of the purchase price, and in return took a
mortgage on the property. One week later, a
bank obtained a judgment against the buyer
for a delinquent credit card balance. The bank
properly recorded its judgment as a lien against
the property. Another month after that, the buyer
incurred some extraordinary medical expenses,
and asked the employer for another $100,000,
which the employer provided and added onto the
principal balance the buyer owed on the loan.
Finally, six months later, the buyer asked the
developer to change the terms of the loan, so
that the buyer would have more time to pay. The
developer and the buyer agreed that the buyer
could have an additional five years to pay the
balance of the loan in exchange for an increase
in the principal of the loan. Shortly thereafter,
the buyer lost his job and defaulted on all of his
payments. The employer brought an action to
foreclose its mortgage. All mortgages and liens
were promptly and properly recorded.
Regarding the distribution of the proceeds of
an eventual sheriff’s sale of the property, which
of the following statements is true?
(A) The bank is paid in full before the developer
is paid in full.
The distribution of sale proceeds in this case would be:
(i) the original amount of the employer’s purchase money
mortgage,
(ii) the bank’s judgment,
(iii) the $100,000 advance by the employer, and finally,
(iv) the amount of the increase in the debt to the developer due to the agreed modification of the principal of the original loan.
The original unmodified purchase money mortgage of the developer would remain on the land because it was senior to the mortgage being foreclosed (the employer’s).
When a mortgagee and an assuming grantee subsequently
modify the original obligation, …
…the original mortgagor is completely discharged of liability.
An owner obtained a loan of $60,000 from a
bank in exchange for a promissory note secured
by a mortgage on his land, which the bank
promptly and properly recorded. A few months
later, the owner obtained another loan of $60,000
from a lender, in exchange for a promissory note
secured by a mortgage on the land, which the
lender promptly and properly recorded. Subsequently,
the owner sold the land to a buyer for
$150,000 and conveyed a warranty deed. The
buyer expressly agreed with the owner to assume
both mortgages, with the consent of the bank and
the lender. A few years later, the bank loaned the
buyer an additional $50,000 in exchange for an
increase in the interest rate and principal amount
of its mortgage on the land. At that time, the
balance on the original loan from the bank was
$50,000. Shortly thereafter, the buyer stopped
making payments on both mortgages and disappeared.
After proper notice to all appropriate
parties, the bank instituted a foreclosure action
on its mortgage, and purchased the property at
the foreclosure sale. At that time the principal
balance on the lender’s mortgage loan was
$50,000. After fees and expenses, the proceeds
from the foreclosure sale totaled $80,000.
Assuming that the jurisdiction permits
deficiency judgments, which of the following
statements is most accurate?
(C) The bank keeps $50,000, the lender is
entitled to $30,000, and only the lender can
proceed personally against the owner for its
deficiency.
The bank’s original mortgage has priority in the proceeds, followed by the lender’s mortgage, and only the lender can proceed against the owner because the bank modified its mortgage after the owner had transferred to the buyer.
Generally, the priority of a mortgage is determined by the
time it was placed on the property, and the proceeds of a foreclosure sale will be used to pay off the mortgages in the order of their priority.
However, if the landowner enters into a modification
agreement with the senior mortgagee, raising its interest rate or otherwise making the agreement more burdensome, the junior mortgage will be given priority over the modification.
Thus, if the first mortgage debt is larger because of the modification, the second mortgage gains priority
over the increase in the debt.
Here, the bank and the buyer modified the original mortgage by increasing the principal amount and the interest rate. This modification is not given priority over the lender’s mortgage, and foreclosure proceeds will not be applied against it because the senior lender’s mortgage was not fully satisfied from the proceeds.
With regard to the deficiency, the owner is liable to the lender because when a grantee signs an assumption agreement, becoming primarily liable to the lender, the original mortgagor remains secondarily liable on the promissory note as a surety.
Here, the buyer assumed the lender’s mortgage and became primarily liable; however, the owner remained secondarily liable as surety and can be required to pay off the rest of the lender’s mortgage loan.
On the other hand, the owner will not be liable to pay off
the balance of the bank’s loan, because when a mortgagee and an assuming grantee subsequently modify the original obligation, the original mortgagor is completely discharged of liability.
The owner had nothing to do with the modification agreed to by the bank and the buyer that increased
the amount of the mortgage debt, and will not be even secondarily liable for that amount.
A grantor may deliver a deed to an escrowee with instructions
that it be delivered to the grantee when certain conditions (e.g., death of the grantor) are met.
When the conditions occur, title passes automatically to the grantee and relates back to the date
of delivery to the escrowee.
Under the estoppel by deed theory,….
….if a grantor purports to convey an estate in property that she does not then own, her subsequent acquisition of title to
the property will automatically inure to the benefit of the grantee, but only as against the grantor.
If the grantor transfers her after-acquired title to a bona fide purchaser for value (“BFP”), the BFP gets good title.
Under the majority view, a recorded deed obtained from
a grantor who had no title at that time but who afterwards obtains title is not in the chain of title and so does not give constructive (or record) notice to a subsequent purchaser.
However, a subsequent purchaser is charged with knowledge of whatever an inspection of the property would have disclosed and anything that would have been disclosed by inquiring of the possessor.
notice statute
last BF-value
Under a notice statute, a subsequent bona fide purchaser (i.e., a person who gives valuable consideration and has no actual, record, or inquiry notice of the prior instrument)
prevails over a prior grantee who failed to record.
A state statute provides as follows:
Any judgment properly filed shall, for
10 years from the date of filing, be a
lien on the real property then owned
or subsequently acquired by any
person against whom the judgment is
rendered.
A landowner conveyed a lot in that state to
his aunt, who had had a judgment lien recorded
against her two years earlier in the county in
which the land was located. One year later, the
aunt conveyed the property to a buyer by general
warranty deed. The deed did not mention the
lien, but the buyer was aware of it. Two years
later, the buyer conveyed the property to a
creditor by special warranty deed. The creditor
was not aware of the lien and her deed also made
no mention of it. One year after that transaction,
the creditor conveyed the property to a developer
by general warranty deed. The developer’s
deed did not mention the lien but the developer
was aware of it. The next year, the developer
entered into a contract to convey the property to
an entrepreneur. The entrepreneur’s title search
disclosed the judgment lien against the aunt,
and the entrepreneur refused to proceed with
the transaction because title was not marketable.
The developer brought an action against the
entrepreneur for specific performance and was
denied relief. He then brought an action against
the aunt, the buyer, and the creditor for breach of
warranty.
Assuming that all transactions concerning the
property were promptly and properly recorded,
and that the party holding the judgment lien has
taken no action as of yet to enforce it, which
parties, if any, will be liable to the developer?
The developer will prevail only against the creditor because only the creditor has committed an actionable breach of the covenant against encumbrances.
A grantor making a conveyance by general warranty deed generally makes five covenants for title, and warrants against title defects created both by herself and by all prior titleholders.
The covenant of seisin, the covenant of right to convey, and the covenant against encumbrances are present covenants and are breached, if at all, at the time of conveyance.
The covenant for quiet enjoyment and the covenant of warranty are future covenants and are breached only on interference with the possession of the grantee or
his successors.
Unlike the future covenants, the present covenants do not “run” with the grantee’s estate and cannot be enforced against the covenantor by successive grantees in most jurisdictions.
Here, because the party holding the judgment lien has not taken any action to enforce it, there is no disturbance of possession and the future covenants have not been breached. Since the only covenant that has been breached was the covenant against encumbrances, only the developer’s
grantor, the creditor, is liable.
In land contracts and deeds, property may be described in various ways; i.e., by reference to a government survey, by metes and bounds, by courses and angles, by references to a recorded plat, by reference to adjacent properties, by the name of the property, or by a street and number system.
When is a description in a deed sufficient?
A description in a deed is sufficient if
it furnishes a good lead as to the identity of the property.
A landowner owned a large tract of mineralrich
land in a sparsely populated area. He
entered into a lease with a prospector who was
interested in developing the land for mining. The
term of the lease was two years and gave the
prospector an option to buy the property at any
time after the first year. The prospector did not
record the lease. Six months later, the prospector
left the land for a period of time to prospect in
Mexico, leaving no goods on the land that would
identify him. The landowner then conveyed the
property in fee simple to a developer, who had
inspected the property while the prospector was
in Mexico and was unaware of the prior transaction.
The developer did not immediately record
her deed. After three months in Mexico, the
prospector returned to the land and encountered
the developer.
A statute in the jurisdiction provides, in part:
“No conveyance or mortgage of an interest in
land, other than a lease for less than one year, is
valid against any subsequent purchaser for value
without notice thereof whose conveyance is first
recorded.”
If the developer brings an action to quiet title,
how should the court rule?
The developer’s ownership of the land is subject to the prospector’s interest because the prospector’s interest was first in time.
The common law rule that priority is given to the grantee who was first in time still applies unless operation of the jurisdiction’s recording statute changes the result.
The statute in this question is a race-notice statute, under which a subsequent bona fide purchaser is protected only if she records her interest before the prior grantee does. While the developer is a bona fide purchaser, she must still win the race to the recording office to prevail over the prospector’s prior interest.
Since neither party has recorded in this fact pattern, the developer will take the property subject to the prospector’s prior interest.
race-notice statute
BF-v who 1st records
a subsequent purchaser for value without notice of any prior conveyance is protected if she records before the prior grantee
A landlord leased an office building to a
tenant for 10 years. The tenant, a secondyear
law student, was familiar with the state’s
recording act, which provided:
No conveyance is valid against any
subsequent purchaser for value without
notice unless the conveyance is recorded.
No lease for three years or more is
valid against a subsequent purchaser
for value without notice unless the
lease has been recorded.
Believing it would be obvious to any prospective
purchaser that the tenant was in possession
of the property, she failed to record the lease.
Shortly thereafter, the landlord entered into
a contract to sell the leased property to a buyer.
Before purchasing the property, the buyer merely
drove by it, and thus did not notice the tenant’s
occupancy. The standard title search did not
reveal the lease because it was unrecorded.
The buyer tendered the purchase money to the
landlord, and the landlord conveyed to the buyer
the property by warranty deed. The buyer subsequently
found the tenant in possession of the
premises and ordered her to vacate. The tenant
refused and asked the buyer where she should
send the rent checks.
In an action by the buyer to evict the tenant,
how should the court rule?
The tenant will prevail because the buyer did not properly inspect the property.
A title search is not complete without an examination of possession. If the possession is unexplained by the record,
the purchaser is obligated to make inquiry. The purchaser is charged with knowledge of whatever an inspection of the property would have disclosed and anything that would have been disclosed by the possessor. Thus, the buyer is on constructive notice of the tenant’s possession and anything
that would have been disclosed by inquiring of the tenant. Because the buyer had this notice, he is not protected by the notice recording statute, and he will take subject to the tenant’s lease.
When a purchaser has paid only part of the purchase price under an installment land contract, most courts hold that the purchaser is protected by the recording acts only to the extent
of payment made.
Depending on the equities involved in the case, the court has three options:
(i) create a tenancy in common in the property, with the contract purchaser receiving a share of the property equal to the proportion of payments made;
(ii) award the land to the prior claimant,
but give the contract purchaser a right to recover the amount she paid (with interest), secured by a lien on the property; or
(iii) award the land to the contract purchaser, but require the contract purchaser to make the remaining payments to the prior claimant. This last obligation is also
secured by a lien on the property
If property
is specifically devised or bequeathed in the testator’s will, but the testator no longer owns that
property at the time of death, the gift
is adeemed; i.e., it fails.
i.e.
Here, the mother specifically
devised the land to the daughter in her will. However, the mother conveyed the same land to the
investor prior to her death. Thus, the land was no longer in the mother’s estate at her death, so the
daughter takes no interest in the land under the will.
The owner of a gourmet food store entered
into oral negotiations with the president of a
food products corporation to secure an exclusive
distributorship in the state for a popular truffle
sauce. After some discussion, the parties agreed
on all salient points and shook hands on the
deal. They agreed further that the corporation’s
general counsel would reduce the agreement to
writing and that the agreement would become
effective after it was drawn up and initialed
by the corporation’s general counsel and by
the store owner’s attorney. The corporation’s
general counsel duly committed the agreement
to writing and sent the writing to the store
owner’s attorney, but without initialing it first.
The attorney looked over the agreement, made
no changes, initialed it, and mailed the agreement to the corporation’s counsel on May 1. On May 2, the president of the corporation decided that a large national grocery chain would be a better distributor for the sauce than the gourmet food store, so he had the general counsel call the store’s attorney to say that the deal was off. The corporation’s counsel received the written agreement with the attorney’s initials on it on May 3. Nevertheless, the corporation began to distribute its products through the national grocery chain. If the gourmet food store owner files suit against the corporation and its president for breach, is she likely to prevail?
(A) Yes, because the mailing of the written
agreement to the attorney constituted an
irrevocable offer.
(B) Yes, because the owner and the corporation’s
president entered into a valid oral
contract.
(C) No, because the corporation’s general
counsel never initialed the written agreement
and there was, therefore, failure of an
express condition.
(B) The gourmet food store owner is likely to prevail, because she and the corporation’s president formed a valid oral contract when they agreed on all salient points after negotiations. One element of their agreement was that the terms would be put in writing, a process sometimes called “memorializing” the agreement. The writing does not constitute the agreement itself, but is merely a written record of it. Another element of their oral agreement concerned when it would take effect: on completion of the “memorialization.” The general counsel’s failure to initial the writing, whether deliberate or inadvertent, was a breach of the oral agreement that he would do so, but cannot be used to avoid performance of the contract. Note that even though the contract involves
the sale of goods and may be for $500 or more, a violation of the Statute of Frauds does not affect the contract’s validity, just its enforceability. Here, the memorialization of the contract makes it enforceable under the Statute (even without his initials).
(A) is incorrect because the oral agreement
was already in effect at the time the writing was mailed. The processes of offer and acceptance
took place during the oral negotiations. Even if the mailing of the writing could somehow
be seen as an offer, there are no facts, such as detrimental reliance, that show it to be irrevocable.
(C) is incorrect because, as described above, the writing was a memorial of the existing oral agreement. Even if the requirement of the general counsel’s initialing is characterized as an
express condition, the corporation’s president would not be permitted to prevent the occurrence of the condition and then claim the benefit of its nonoccurrence.
The order department of a machine tools manufacturing company received a phone call
from a factory owner who placed an order for two of the company’s standard “Type-A” machines. The factory owner and the company came to an oral agreement whereby the total price for both machines was agreed to be $10,000. The first machine was to be delivered on May 1, with payment of $5,000 due 30 days after delivery, and the second machine was to be delivered on June 1 on the same terms (payment of $5,000 due 30 days after delivery). Although the company did not carry the machine in stock,
no retooling was required because the Type-A
machine was a standard model. The first machine was duly delivered on May 1. The second machine arrived on June 1, but the factory owner refused to accept delivery and also refused to pay for the first machine. The company sued the factory owner on June 2. Assume that it cost the company $3,000 to manufacture each Type-A machine, and that the company could resell the machine for only $3,000. What damages should be awarded aside from any incidental damages?
(A) $3,000.
(B) $5,000.
(C) $7,000.
(D) $10,000.
(B) The company should recover $5,000 because the oral contract between it and the factory owner is enforceable to the extent the factory owner received and accepted the goods.
A promise for the sale of goods of $500 or more is not enforceable unless evidenced by a writing signed by the party to be charged. [UCC §2-201(1)] However, an oral contract for such goods is enforceable to the extent of goods received and accepted by the buyer. [UCC §2-201(3)(c)] Oral contracts for specially manufactured goods not suitable for sale in the ordinary course of the seller’s business also are enforceable when the seller has begun substantially to perform.
Here, the parties’ agreement was oral. The factory owner accepted one machine, but neither machine was specially manufactured. He is bound to pay the $5,000 contract price for the accepted machine but is not bound to pay for the rejected machine.
(A) is incorrect because the company is entitled to the contract price for the machine the factory owner accepted, not just restitution. If a contract is unenforceable because of noncompliance with the Statute of Frauds, a party can generally sue
for the restitution of any benefit that has been conferred. However, as discussed above, an oral
contract for the sale of goods of $500 or more is enforceable to the extent of goods received and
accepted by the buyer. Therefore, because the factory owner accepted one machine, the company is entitled to the $5,000 contract price of that machine, not just $3,000 in restitutionary damages.
(C) is incorrect because the company is not entitled to any damages as to the rejected machine. As indicated above, the company is entitled to $5,000 for the first machine. There is no enforceable contract regarding the second machine, and the company is not entitled to damages for that machine. If there were an enforceable contract for the second machine, (C) would state a proper measure of damages—if a buyer breaches by refusing to accept goods, the seller is entitled to recover the difference between the contract price ($5,000) and the market or resale price ($3,000), here, $2,000. Thus, the company would be entitled to $5,000 for the accepted machine and $2,000 for the rejected machine, or $7,000.
(D) is incorrect for the same reason that (C) is incorrect— the company is only entitled to the contract price of the machine accepted by the factory owner. If the contract for the second machine were enforceable, (D) would still not be a proper measure of damages. Under the UCC, the seller has a right to force goods on a buyer who has not accepted them only if the seller is unable to resell the goods or if the goods have been lost or damaged after the risk of loss passed to the buyer. [UCC §2-709] Because the company can resell the second machine, which has not been lost or damaged, it could not recover the full price of the second machine from the factory owner even if the contract was fully enforceable.
A leading computer supply retailer contacted
a manufacturer of flash drives on October 25
to supplement the supply of flash drives at the
retailer’s area stores. In response, the manufacturer offered to supply 50 boxes of flash drives at $200 a box, for a total price of $10,000,
delivery one week after acceptance, and sent
a letter containing those contractual terms to
the retailer on October 26. The president of the
retailer signed the letter on October 27 without
making any changes to it and sent it back to the
manufacturer by first class mail. The next day,
having not heard from the retailer, the manufacturer’s sales manager contacted the president of the retailer and informed him that, if the retailer accepted by the end of the month, it would receive a 1% discount on the total price. When the retailer’s president responded that he had already sent the acceptance, the sales manager assured him that the discount would still apply. On October 31, the retailer received a circular from a competing source offering comparable flash drives for 5% less than the manufacturer’s price. The retailer immediately faxed a rejection to the manufacturer. The manufacturer received the rejection immediately but took no action on it. The next day, November 1, the manufacturer received the signed contract from the retailer. What is the status of the parties’ agreement?
(A) An enforceable contract was formed for
$9,900 because the retailer accepted before
October 31 and needed no additional consideration
for the oral modification.
(C) An enforceable contract was formed for
$10,000 because, even though the retailer
accepted before October 31, the Statute
of Frauds makes the oral modification
between the parties unenforceable.
(C) A contract was formed for $10,000 because the oral modification is unenforceable under the
Statute of Frauds.
Under the UCC, a contract for the sale of goods priced at $500 or more is not enforceable unless evidenced by a writing. Furthermore, contract modifications must also meet the Statute of Frauds requirement if the contract as modified is within the Statute’s provisions.
Here, the Statute of Frauds is applicable to both the original contract and the contract as modified.
Since the modification was not in writing, it is not enforceable; hence, the terms of the original
contract, which satisfies Statute of Frauds requirements, are effective.
A gambler lived in a state where gambling was illegal. Nevertheless, he gambled on a regular basis. He asked his friend to lend him $5,000 to bet on a football game. His friend agreed to lend him $5,000 if the gambler would bet half of it on the friend’s behalf. The gambler agreed, took his friend’s $5,000, and placed the bet. The gambler won on 4-to-1 odds. He gave his friend his $5,000 back but refused to tender any winnings.
If his friend sues the gambler to recover the winnings due under the contract, who will
prevail?
(A) His friend, because he fully performed his
part of the bargain.
(B) His friend, because the court will not allow
the gambler to unfairly profit from his
illegal contract.
(C) The gambler, because the contract was
illegal and the court will not enforce an
illegal contract.
(D) The gambler, because the contract was
illegal and the court will only act to put the
parties in the status quo ante, and his friend
already has his money back.
(C) The gambler will prevail.
The general rule is that a court will not enforce a contract if its subject matter or consideration is illegal; the court will leave the parties as it finds them.
Here, the subject matter of the contract, placing gambling bets, is illegal in the state. Thus, (A) is wrong because his friend’s performance is irrelevant. (B) is wrong because the court will refuse to help either party to an illegal contract, even where one party has gained unfairly. (D) is wrong because the court will not put the parties back into the position they were in prior to entering into the contract, but rather will leave them where they stand.
Mutual mistake of an existing material fact to a contract is a defense that allows the adversely affected party to …
…rescind as long as that party did not assume the risk of mistake.
***
Mutual mistake arises where both parties are mistaken as to an existing fact relating to a basic
assumption of the contract and the mistake has a material effect on the agreed-upon exchange.
Where only one of the parties entering into a
contract is mistaken about facts relating to the agreement, this unilateral mistake will not prevent
formation of a contract unless ….
…the nonmistaken party is or had reason to be aware of the mistake made by the other party.
On reaching majority, an infant may affirm; that is, choose to be bound by her contract.
When a voidable promise is reaffirmed, the promise will be enforced according to the terms of the reaffirmation rather than the original obligation.
A woman contracted with a dog breeder for the purchase of a show dog. Prior to the sale, the
breeder falsified the dog’s pedigree records to
indicate that there were several champions in the
dog’s bloodline. After a few weeks of enjoying
the dog’s companionship, the woman discovered
the breeder’s actions when she attempted to register the dog with a local dog club. The woman wishes to keep the dog, but is understandably unhappy with the high price that she paid. What is the woman’s best course of action for relief?
(A) The woman should notify the seller and sue
for warranty damages.
(B) The woman should seek to rescind the
contract based on fraudulent misrepresentation.
(C) The woman should seek reformation of the
contract to reflect a fair price for the dog.
(D) The woman should take no action because
she wishes to keep the dog.
(A) The woman should notify the seller and sue for damages because the dog the woman received was not of the quality she was promised, in violation of an express warranty made by the seller.
Any affirmation of fact made by a seller to a buyer creates an express warranty if the statement
is part of the basis of the bargain.
Here the woman was looking for a show dog, and the breeder represented to her that this dog was part of a champion bloodline. The dog’s pedigree clearly was a part of the basis of the bargain because the buyer paid a higher price for the dog than she would have if the dog was not from a champion bloodline. The breeder’s actions created an express warranty. The dog the woman received, however, was not as the breeder represented, thus the breeder breached this warranty. Nevertheless, the woman has decided to keep the dog. If a buyer accepts goods that breach one of the seller’s warranties, the buyer may recover damages. The basic measure of damages in such a case is the difference between the value of the goods as delivered
and the value they would have had if they had been according to contract, plus incidental and
consequential damages. To recover damages for any defect as to accepted goods, the buyer must,
within a reasonable time after she discovers or should have discovered the defect, notify the seller of the defect. This would be the best course of action among the options given for the woman because she would keep the dog and get back some of the money she overpaid for the dog.
(B) is not the correct answer. While it is true that the woman could rescind the contract based on the seller’s fraudulent misrepresentation, it is not the best option in this case. Rescission is a remedy whereby the original contract is considered voidable and rescinded. The parties are left as though a contract had never been made. The woman here does not want to avoid the contract entirely since she wants to keep the dog.
(C) is incorrect. Misrepresentation as to the subject matter of an agreement is not grounds for reformation because a court will not remake the parties’ bargain. Rescission and/or damages are the proper remedies for misrepresentation.
(D) is incorrect because it is not the best option for the woman to get relief. As discussed above, a buyer can decide to keep nonconforming goods and still recover damages.
A gallery owner offered to sell a piano to an acquaintance for $400. The acquaintance had been to the gallery owner’s house a few
weeks prior to the offer and had seen a Steinberg piano in his living room, so she accepted. Unbeknownst to the acquaintance, the gallery owner also owned a Hairwin piano, which he kept at the gallery, and that was the piano that
he intended to sell. The acquaintance had never
been to the gallery, and the gallery owner was
aware that the acquaintance had never seen the
Hairwin. If the acquaintance sues the gallery owner to obtain the Steinberg, which party is likely to prevail?
(A) The acquaintance, because the gallery
owner knew of the ambiguity.
(B) The acquaintance, because that is the result
under the objective test.
(C) The gallery owner, because there was a
mutual mistake.
(D) The gallery owner, because he subjectively
intended to sell the Hairwin instead of the
Steinberg.
(A) The acquaintance will likely prevail because the gallery owner knew of the ambiguity.
Contract language with more than one possible meaning leads to different results depending on the awareness of the parties. If one party was aware of the ambiguity and the other party was not at the time of contracting, a contract will be enforced according to the intention of the party who was unaware of the ambiguity.
Here, the gallery owner knew he had two pianos, and the acquaintance did not. Thus, there is a contract on the terms as understood by the acquaintance; there is a contract for the Steinberg.
The doctrine of waste governs the obligations between a life tenant and the holder of the remainder regarding the payment of a mortgage on the property. Under this doctrine, …
… a life tenant is obligated to pay interest on any encumbrances on the land, but he does not have to pay anything on the principal of the debt; reversioners or remaindermen must pay the principal in order to protect their interests.
There are three types of restraints on alienation:
(i) disabling restraints, under which any attempted transfer is ineffective;
(ii) forfeiture restraints, under which an attempted transfer results in a forfeiture of the interest; and
(iii) promissory restraints, under which an attempted transfer breaches a covenant.
Any total restraint on a fee simple—either forfeiture, disabling, or promissory—is void.
A rancher deeded his ranch “to my son so
long as he lives, and on his death to my son’s
widow for the remainder of her life, and on her
death to each of her lineal descendants who are
living on her death, to share and share alike.”
The deed was validly executed, delivered, and
recorded. At the time of the conveyance, the
son was married but had no children. Three
years later, the rancher died. His will left all
of his estate to his sister. Two years after the
rancher’s death, the son was killed in an automobile accident, leaving his widow with twin
girls. Upon the widow’s death many years later,
she was survived by both girls, her only lineal
descendants. Who now owns the ranch?
(A) The girls are tenants in common, with each
owning an undivided one-half; their interest
vested on the son’s widow’s death, well
within the period of lives in being plus 21
years.
(B) The girls are tenants in common, with
each owning an undivided one-half; they
received a vested remainder, which is not
subject to the Rule Against Perpetuities.
(C) The sister owns the ranch in fee simple,
because the Rule Against Perpetuities
voided the gift over to the son’s widow and
the following gift to her lineal descendants.
(D) The sister owns the ranch in fee simple,
which she received upon the death of the
son’s widow.
(D) The sister owns the ranch, because the gift over to the son’s widow’s descendants was void.
This is the case of the unborn widow. Remember, the Rule Against Perpetuities is measured and enforced at the time of the grant, not later. If there is any way, at the time of the grant, that a gift over may vest later than any life in being plus 21 years, it is void.
Take the grants one at a time, in order.
The gift to the son is valid; he was alive at the time of the grant.
The gift to the son’s widow also is valid. Although her life estate is a contingent remainder (because we would not know the identity of the son’s widow until the son dies), we would know her identity (and thus the remainder will become vested) at the moment of the son’s death, clearly within the time period of the Rule.
But the gift to the widow’s descendants violates the Rule Against Perpetuities. The son’s widow was not necessarily born and in existence at the time of the conveyance, and we will not know who the widow’s descendants are until she dies, which may be well outside the lives in being at the time of the rancher’s transfer, plus 21 years. Since the gift to the widow’s descendants
is void, the grant effectively reads: “to my son for life, then to my son’s widow for life.”
That would have left a reversion in the rancher, which the sister inherited under the rancher’s
will. The fact that the son’s widow was in fact alive at the time of the rancher’s grant does not
matter because the Rule Against Perpetuities is applied at the time of the grant. Subsequent facts
are irrelevant.
A co-tenant has no duty to improve the
property and cannot force the other co-tenants to contribute to the cost of improvements made
by him. Can the value of improvements ever be recouped?
Only in an action for partition can the value of improvements be recouped.
By mutual agreement, a brother and sister
purchased a 10-acre parcel of land and took
title “as joint tenants with right of survivorship.”
Three years after the purchase, the brother asked if he could build an apartment house on his half of the property; the sister agreed. He then built an apartment house on the eastern five acres of the property. Two years later, the brother died, leaving his entire estate to his son. In an action for partition, what is the best argument in favor of the brother’s estate being judged the owner of the eastern five acres?
(A) The sister’s conduct during the brother’s
lifetime estopped her from asserting title to
the eastern half of the property.
(B) The taking of title as joint tenants does not
conclusively presume that the property is
held as joint tenants.
(C) The joint tenancy was terminated at the
time the oral agreement was made.
(D) A joint tenant may will away his interest in
property to a lineal descendant
(A) Because the sister informed her brother that he could develop the eastern five acres of the land, and because he reasonably relied on her statement to his detriment, she may be estopped to deny her brother’s likely intended effect, i.e., to develop the property and pass on the benefit of
the improved portion to his son.
(B) is incorrect. Where the language of the grant is clear and a joint tenancy is created, no presumption is needed or applies. The joint tenancy cannot later be changed by subsequent informal action.
(C) is wrong because an oral agreement is not effective to terminate a joint tenancy.
(D) is an incorrect statement of the law. A joint tenant may not pass an interest to anyone at death, due to the right of survivorship.
A landowner devised her parcel of land to her
daughter, her heirs, and assigns, “so long as the
property is used for residential purposes, then to
my niece, her heirs, and assigns.” The remainder
of the landowner’s property passed through the
residuary clause of her will to her grandson. The
daughter lived on the land for 25 years; then, on
her death, ownership passed to her husband. In
the meantime, the niece had also died, leaving
her entire estate to her son. The husband has
leased the land to a developer, who has obtained
the necessary permits to build a shopping center
on it. The grandson and the niece’s son both file quiet title and ejectment actions against the
husband, and the cases are consolidated. How should the court, applying common law, rule as to ownership of the land?
(A) For the husband.
(B) For the niece’s son.
(C) For the grandson, because he received a
right of reversion from his grandmother.
(D) For the grandson, because he received
a possibility of reverter from his grandmother.
Remember! Executory interest following defeasable fee violates RAP.
(D) The court should rule for the grandson because he received a possibility of reverter from his grandmother. The landowner attempted to give the daughter and her successors a fee simple subject to an executory interest, with the niece and her successors holding the executory interest. However, the attempted gift to the niece and her successors fails under the Rule Against Perpetuities because the niece’s interest could vest in possession more than 21 years after a life in being.
Thus, (B) is incorrect. After the void interest is stricken, the daughter and her successors have a
fee simple determinable and the landowner retained a possibility of reverter, which passed to the grandson through the residuary clause in the landowner’s will. When the husband, the daughter’s successor, ceased using the property for residential purposes, the possibility of reverter matured, leaving ownership in the grandson. (A) is therefore incorrect. (C) is incorrect because a possibility of reverter, not a right of reversion, is the interest left in the grantor when a fee simple determinable is created.
A college student assigned her apartment
rental lease, by written agreement, to one of
her sorority sisters. The sorority sister, in turn,
assigned the lease to a classmate. If the classmate fails to pay the rent, can the landlord bring suit against the sorority sister to recover this money?
(A) Yes, because the sorority sister remains in
privity of estate with the landlord.
(B) Yes, because the sorority sister remains in
privity of contract with the landlord.
(C) No, because the sorority sister is no longer
in privity of estate with the landlord.
(D) No, because the fact that the college student
was allowed to assign the lease means that
the sorority sister is allowed to assign it
also.
L — T (priv of K) — A — A (priv of estate)
(C) The landlord cannot bring suit against the sorority sister. Absent an express assumption, an
assignee is not liable on the original covenants once she reassigns. Hence, (A) is wrong.
(B) is wrong because an assignee is not in privity of contract with the lessor unless the assignee
expressly assumes the lease obligations.
(D) is incorrect because the sorority sister’s ability
to subsequently assign the lease is unrelated to the issue of liability. The sorority sister, as an
assignee of the college student, is liable for covenants running with the land on the basis of privity of estate. Once the sorority sister assigned the lease to her classmate, the privity of estate ends and the sorority sister’s liability is terminated.
***
An assignee is in privity of estate with the lessor, and is liable for those covenants in the original lease that run with the land.
An attorney was staying at a hotel while traveling to work on a case. The normal room charges were $150 per night. The attorney made a deal with the hotel management to pay $700 per week for his room for an indefinite period. The attorney never told the hotel management exactly how long he planned to stay or how he
would pay, but each Sunday he would present the cashier with a check for $700, plus any additional taxes and room charges, to pay for the previous week’s lodging. He did this for four weeks, and then the case he was working on was
settled. On a Thursday morning, before checkout
time for that day, the attorney tendered the
hotel cashier a check for $400 plus additional
charges incurred from Sunday through Wednesday nights. The cashier promptly took
the check and then demanded that the attorney
pay an additional $1,000 plus taxes for 10 more
days’ lodging, covering the rest of the week
plus an additional week because he did not give
a week’s notice that he planned to vacate. The
attorney refused to pay the additional charges. If
the hotel sues the attorney and the court determines that the parties created a tenancy, for
how many additional days’ lodging will he be
required to pay?
(A) None, because a tenancy at will was created.
(B) Three days, because a periodic tenancy was
created.
(C) Seven days, because a periodic tenancy was
created.
(D) Ten days, because a periodic tenancy was
created.
(D) The attorney must pay 10 days’ lodging because his notice to terminate will not become effective until then.
In the usual case, a hotel guest is treated as a licensee rather than a tenant. Here, however, the court found that the parties created a tenancy because they specifically agreed to a week-to-week arrangement at a special weekly rate.
A periodic tenancy arises when the parties
do not fix the duration of the tenancy. It continues from period to period and is automatically renewed for another period until terminated by the giving of proper notice.The period is generally based on an express understanding between the parties or an implied understanding based on the payment of rent.
The attorney’s arrangement with the hotel constitutes a periodic weekto- week tenancy because it was of indefinite duration and “rent” was paid each week.
To terminate a periodic tenancy of less than one year, a full period in advance of the period in question is required by way of notice. For a week-to-week tenancy, notice at least one full week prior to vacating would have to be given prior to the beginning of the period. Thus, the attorney’s notice had no effect on his liability for that week’s rent and would operate as one week’s notice starting at the beginning of the next week.
(A) is incorrect. If the parties had expressly agreed that either party could terminate at any time, a tenancy at will would have been created and the attorney would be liable for nothing. However, the payment of rent on a regular periodic basis will cause a court to treat the tenancy as a periodic tenancy.
(B) is incorrect because it does not take the one
week’s notice requirement into account.
(C) is incorrect because the notice must fix the last day of the period as the date of termination rather than some intervening day. Thus, the notice period does not begin to run until the end of the current week.
A landlord’s promise in a lease to maintain the property does not terminate because the property is sold.
Although no longer in privity of estate, the original landlord and tenant remain in privity of contract, and the original landlord remains liable on the covenant unless there is …
…a novation.
A novation substitutes a new party for an original party to the contract. It requires the assent of
all parties, and completely releases the original party.
A landlord leased an apartment to a tenant
for five years. The lease provided that the
landlord will: (i) keep the apartment building at
a comfortable temperature 24 hours per day, and
(ii) have the carpets cleaned once a year. Two
years later, the landlord began turning off the air
conditioning at 10 p.m. The tenant’s apartment
became hot and stuffy, and she demanded that
the landlord honor the covenant. The landlord
refused. The following month, the pipes burst
in the tenant’s only bathroom, rendering it
unusable. The resultant flooding soiled some
of the carpeting, which had not been cleaned
in the past 12 months. The tenant reported the
problems to the landlord, who did not return the
tenant’s phone calls.
Which of the following are valid reasons for
the tenant to terminate the lease?
(A) Only that the landlord did not keep the
apartment building at a comfortable temperature
24 hours per day.
(B) Only that the landlord did not fix the
bathroom pipes.
(C) That the landlord did not keep the apartment
building at a comfortable temperature
24 hours per day and did not fix the
bathroom pipes.
(D) That the landlord did not keep the apartment
building at a comfortable temperature
24 hours per day, did not have the carpets
cleaned, and did not fix the bathroom pipes.
(B) The tenant will be successful in terminating the lease because the landlord breached the implied warranty of habitability by failing to fix the bathroom pipes.
The general rule at common law was that the landlord was not liable to the tenant for damages caused by the landlord’s failure to maintain the premises during the period of the leasehold. Today, however, a majority of jurisdictions,
usually by statute, provide for an implied warranty of habitability for residential tenancies.
_In the absence of a local housing code, the standard applied is whether the conditions are
reasonably suitable for human residence._If the landlord breaches the implied warranty, the tenant may: (i) terminate the lease, (ii) make repairs and offset their cost against future rent, (iii) abate rent, or (iv) seek damages.
Here, a court is likely to consider the lack of a functioning bathroom as making the premises unsuitable for human residence, allowing the tenant to terminate the lease. (A) is therefore incorrect.
(C) would be a stronger answer if the tenant had vacated the premises within a reasonable time. The doctrine of constructive eviction provides that where a landlord does an act or fails to perform some service that he has a legal duty to provide, and thereby makes the property uninhabitable, the tenant may terminate the lease and seek damages. However, a tenant cannot claim a constructive eviction unless: (i) the injurious acts were caused by the landlord, (ii) the premises are uninhabitable, and (iii) the tenant vacates the premises within a reasonable time. Here, the landlord’s failing to keep the apartment building at a comfortable temperature 24 hours per day meets conditions (i) and perhaps (ii), but the tenant remains in possession. Therefore, the tenant cannot claim constructive eviction and (C) is incorrect. (D) is incorrect for the same reason.
A developer created an exclusive residential
subdivision. In his deed to each lot, the following
language appeared:
Grantee agrees for himself and assigns
to use this property solely as a singlefamily residence, to pay monthly fees as levied by the homeowners’ association for upkeep and security guard services, and that the backyard of this property shall remain unfenced so that bicycle paths and walkways may run through each backyard, as per the subdivision master plan [adequately described], for use by all residents of the subdivision.
The developer sold lots to an actuary, a baker,
and a coroner. All deeds were recorded. The
subdivision was developed without backyard
fences, with bicycle paths and walkways in
place in accordance with the general plan. The
actuary in turn sold to an accountant by a deed
that omitted any mention of the covenants above,
and the accountant had no actual knowledge
thereof. Shortly thereafter, the accountant started
operating a tax preparation business out of his
home. The baker in turn sold to a barber, who
knew of, but refused to pay, the monthly fees
levied by the homeowners’ association. The
coroner leased her property for 10 years to a
chiropractor, who erected a fence around the
backyard, unaware of the covenant against such
fencing.
According to common law principles, which
of the following statements is correct?
(A) If the developer, still owning unsold lots,
sues the accountant to have him cease
operating the tax preparation business, the
accountant would win because there is no
privity between the developer and the accountant.
(B) If the homeowners’ association sues the
barber to collect the monthly fees for
upkeep and security guard services, the
homeowners’ association would win
because the covenant regarding fees is
enforceable in equity against the barber.
(C) If the barber sues the chiropractor to obtain
removal of her backyard fence, the barber
would win because the covenant regarding
fencing is enforceable in equity against the
chiropractor.
(D) If the chiropractor sues the accountant to
have him cease operating the tax preparation
business, the chiropractor would win
because the covenant regarding singlefamily
use is enforceable at law against the
accountant.
An equitable servitude is a covenant that, regardless of whether it runs with the land at law, equity will enforce against the assignees of the burdened land who have notice of the covenant.
The benefit of an equitable servitude runs to successors if: (i) the original parties so intended, and (ii) the servitude touches and concerns the land.
The burden runs if (i) and (ii) are met and (iii) the
subsequent purchaser has actual or constructive notice of the covenant.
Privity of estate is not needed to enforce an equitable servitude because it is enforced not as an in personam right against the owner of the servient tenement, but as an equitable property interest in the land itself.
Here, the original parties intended for the fencing covenant to be enforceable by and against assignees, as shown by the specific language of the covenant (“Grantee agrees for himself and assigns”) and its purpose to provide bicycle paths and walkways running through each backyard for the use of all subdivision residents.
The benefit of the covenant touches and concerns the barber’s property because it increases his enjoyment thereof by providing him with such paths and walkways. Therefore, the barber is entitled to enforce the covenant.
The burden of the covenant touches and
concerns the land occupied by the chiropractor because it restricts the landholder in her use of
the parcel (i.e., her rights in connection with the enjoyment of the land are diminished by being
unable to fence in the backyard).
The chiropractor will be deemed to have inquiry notice of the restriction because the subdivision is sufficiently developed in accordance with a general plan for the subdivision.
Moreover, any neighbor in a subdivision can enforce a covenant contained in a subdivision deed if a general plan existed at the time he purchased his lot. As has been noted, the maintenance of access to all backyards for use as bike paths and walkways was part of such a general plan.
Finally, the fact that the chiropractor did not succeed to the coroner’s entire estate,
but rather a leasehold interest, is irrelevant because privity is not required to enforce an equitable
servitude. Therefore, all of the requirements are in place for the existence of an equitable servitude,
which can be enforced by the barber against the chiropractor. (A) is incorrect because
there is privity between the developer and the accountant. There was horizontal privity between
the original covenanting parties because, at the time the actuary entered into the covenant with
the developer, they shared an interest in the land independent of the covenant (i.e., they were in
a grantor-grantee relationship). The accountant holds the entire interest held by the actuary at
the time the actuary made the covenant; thus, there is vertical privity. (B) is incorrect because
the remedy sought is the payment of money. Breach of a real covenant, which runs with the
land at law, is remedied by an award of money damages, whereas breach of an equitable servitude
is remedied by equitable relief, such as an injunction or specific performance. Because the
homeowners’ association seeks to obtain from the barber the payment of money, it is inaccurate to
refer to this as a situation involving an equitable servitude. (D) is incorrect because, as explained
above, if equitable relief is sought, the covenant must be enforced as an equitable servitude rather
than a real covenant.
The doctrine of reciprocal negative servitudes.
When a developer subdivides land into several parcels and some of the deeds contain
negative covenants, but some do not, negative covenants or equitable servitudes, binding all of the parcels in the subdivision, may be implied under the doctrine of reciprocal negative servitudes.
To enforce a reciprocal negative servitude, the court will need to find:
_(i) a common scheme for
development, and_
(ii) notice of the covenants.
A common scheme may be evidenced by a general pattern of prior restrictions.
Notice may be _a_ctual (i.e., direct knowledge of covenants in prior deeds), _i_nquiry (i.e., neighborhood appears to conform to common restrictions), or _r_ecord (i.e., prior deeds containing covenants are in grantee’s chain of title).
When the owner had a scheme for an exclusively residential subdivision that included lots when the sales began (as evidenced by the recorded plat), a court will imply a reciprocal negative servitude limiting the remaining lots to the same use.
i.e.
Owner had 250 lots. Sold 175 with negative covenants. Sold remaining 75 to a different buyer at a later time.
A developer divided his tract of land into four
standard lots, which he conveyed to a doctor, a
pilot, a carpenter, and an athlete, respectively.
Each deed granted by the developer contained
a covenant requiring that the property be used
only for single-family housing. All deeds were
duly recorded in the office of the county recorder
of deeds. The doctor and the pilot proceeded
to build single-family houses on their lots. The
carpenter and the athlete did not develop their
properties immediately.
The doctor later sold her property to a nurse
and included the covenant limiting use to singlefamily dwellings in the deed.
The pilot sold his property to a flight attendant, but did not include the covenant in the deed.
The carpenter sold her property to an electrician, and the deed contained the restriction.
The athlete sold his property to a physical therapist, and the deed did not contain the restriction.
All but the electrician’s deed were duly recorded.
Subsequently, the nurse died and her property
passed by will to her daughter.
The flight attendant gave her land to her son “for life.”
The electrician sold his property for value to
a plumber.
All three transfers of title were recorded, but none of the deeds mentioned the covenant.
Which of the current owners below is not bound by the covenant?
(A) The nurse’s daughter, who received the
property by will.
(B) The flight attendant’s son, who received the
property for life.
(C) The plumber, who purchased the property
from the electrician.
(D) The physical therapist, who purchased the
property from the athlete
(B) The covenant may not be enforced at law against the son.
If all requirements are met for the burden of a covenant to run, the successors in interest to the burdened estates will be bound by the arrangement entered into by their predecessors as effectively as if they had expressly agreed
to be bound.
The requirements are:
(i) The covenanting parties must have intended that successors in interest to the covenantor be bound by the terms of the covenant. The requisite intent may be inferred from circumstances surrounding creation of the covenant.
This requirement is satisfied because the developer’s deed to each of the grantees contained a covenant requiring that the property be used only for single-family housing.
(ii) By virtue of the recording statutes, a subsequent purchaser of the promisor’s land must have actual, inquiry, or constructive (record)
notice of the arrangement at the time she purchased the land; otherwise, she is not bound.
Here, the daughter, the son, and the physical therapist have at least constructive notice of the restriction because it is in their chain of title. Because the electrician’s deed was never recorded, the plumber had a duty to inquire of the electrician where he obtained the property. Thus, the notice requirement is met.
(iii) The covenant must “touch and concern” the land; i.e., the performance of the burden must diminish the landowner’s rights, privileges, and powers in connection with her enjoyment of the land. The current owners’ rights as landowners are diminished because they cannot use their land to construct multifamily dwellings.
(iv) Finally, there must be horizontal and vertical privity.
Horizontal privity requires that, at the time the promisor entered into the covenant with the promisee, the two shared some interest in the land independent of the covenant.
The developer and each of the four original owners, as grantor and grantees, shared an interest in the land independent of the covenant.
Vertical privity exists when the successor in interest to the covenanting party holds the entire interest that was held by the covenantor at the time she made the covenant.
Here, the daughter, the plumber, and the physical therapist took the entire interest (fee simple absolute) from their predecessors. However, the son possesses only a life estate in the property, which is less than the fee simple absolute held by the flight attendant. Thus, vertical privity is lacking and the son cannot be bound by the covenant.
(A) is incorrect because the daughter is bound by the covenant as discussed above. The fact that she took the property by will is irrelevant. In fact, taking in this manner increases the chance that the daughter will be bound because it means she is not a bona fide purchaser for value and thus not protected by the notice requirement of the recording act.
(C) is incorrect because the plumber also is bound because he is charged with inquiry notice of the electrician’s unrecorded deed, which contained the singlefamily restriction.
(D) is incorrect because even though the physical therapist’s deed did not contain the restriction, she will be charged with constructive notice of it. The restriction appears in the developer’s deed to the athlete and thus is in the physical therapist’s chain of title. Hence, she also is bound by the covenant.
A homeowner owned a parcel of land on which she built a single-family residence. To pay for the construction, she obtained financing from a bank in exchange for a mortgage on the land. The bank promptly and properly recorded its mortgage. When the house was completed, except for the absence of an oven in the kitchen,
the homeowner leased the house to a tenant for
a three-year term. There was no provision in the lease agreement regarding kitchen appliances.
The homeowner bought an oven from an appliance company and had it installed in the space provided around the built-in cabinets in the
kitchen. To make the purchase, the homeowner
signed a security agreement with the appliance
company granting it a security interest in
the oven in exchange for financing. The appliance company did not file or record its security interest in the oven.
By the end of the lease term, the homeowner
was in serious default on her mortgage payments
to the bank and to the appliance company. In
preparing foreclosure proceedings against the
homeowner, the bank learned that the tenant was
planning to remove the oven and take it with
him when he moved out within the next few
weeks. The bank filed an action against the tenant claiming ownership of the oven, and joined the homeowner and the appliance company as parties.
Which party has a superior claim to the oven?
(A) The bank, because its mortgage interest
attaches to all fixtures on the real estate and
it has priority over the appliance company.
(B) The tenant, because removal of the oven
will not cause substantial damage to the
real estate.
(C) The homeowner, because the oven
was annexed to the real estate after the
mortgage was given.
(D) The appliance company, because it has
a valid security interest in the oven even
though it was not recorded.
(A) The bank will win because it recorded its mortgage on the property.
In a fixture case involving common ownership, the majority rule is that the annexor’s-intention test applies regardless of whether the owner makes the annexation before or after mortgaging the land.
The mortgage attaches to all fixtures on the real estate in the absence of an agreement to the contrary.
As between a mortgagee and the holder of a security interest in chattel affixed to the land, whichever interest is first recorded in the local real estate records wins.
While a purchase money security interest can gain priority if it is recorded within 20 days after affixation, the appliance company never filed.
(B) is wrong because it states the standard for divided ownership cases, i.e., cases in which the person who brings the chattel onto the land does not own the land. Here, the tenant has no interest in the oven; the competing interests are those of the homeowner and the creditors.
(C) is wrong because, as discussed above, the mortgagee’s interest applies to the homeowner’s fixtures regardless of the time of affixation.
(D) is wrong because, as discussed above, the appliance company did not record a fixture filing.
A landlord leased a vacant commercial
building to a tenant for a 10-year term. On
taking possession, the tenant installed a bar,
booths, special lighting, and a raised dance
floor. The bar and booths were simply placed on
the floor and were not secured to it. The lights
were installed by a qualified electrician and
were directly wired into the building’s electrical
system. The special dance floor was bolted to
the building’s cement floor by installation crews
from the company that made the dance floor.
After installation of the above, the tenant then
operated her business for almost 10 years. At
that time, she decided that business was so good
that she would move to a larger space down the
street. She told the landlord that she would not
be renewing her lease and that she would be
removing the lights, booths, bar, and dance floor.
The landlord told her that none of the installations
could be removed because they are now
part of the building.
If a court were called upon to resolve this
dispute, how would it likely rule?
(A) The tenant may remove the dance floor,
bar, and lights, but not the booths.
(B) The tenant may remove the bar and the
booths, but not the dance floor and the
lights.
(C) The tenant may remove the bar, booths,
dance floor, and lights.
(D) The tenant may not remove the bar, booths,
dance floor, or lights.
(C) All of the items listed may be removed either because they were not affixed to the real property or because they were used in the tenant’s business and thus may properly be termed trade fixtures. As long as the tenant can remove trade fixtures with little damage to the real property,
the tenant may always take them when the tenant moves.
(A) is incorrect. For a chattel to be deemed a fixture (and thus remain as a part of the realty) it must be affixed to the real property.
The booths, along with the bar, were not even attached to the realty; thus, they cannot under any circumstances be considered fixtures, and the tenant may always take them.
(B) is incorrect. If these were not trade fixtures, an argument could be made that the tenant could take the items not affixed (bar and booths) but not the ones affixed (lights and dance floor). But even items strongly attached to the realty may be detached by the tenants who installed them if, as here, they are tools of the trade.
(D) is incorrect. The booths and bar can be removed because they were not affixed
to the building, and even though the lights and dance floor were attached, they can be removed as trade fixtures.
An easement can be abandoned provided two things are satisfied:
- nonuse of the easement
- together with some affirmative act taken on the property which would indicate an intent to abandon the easement.
- Mere nonuse would not be enough, no matter how long the easement was not used. It is the act taken on the property that shows the intent to abandon that is crucial.
…. , has the right to choose the location of an easement by necessity.
The holder of the servient estate
The owner of the servient parcel has the right to locate the easement, provided the location is reasonably convenient.
The doctrine of equitable conversion places the risk of loss on …. as soon as the enforceable contract is entered into.
the purchaser
The fee simple owner of an unimproved
parcel of wooded land orally agreed to sell it
to a buyer under an installment land contract.
The buyer agreed to pay $5,000 down and $100
a month for the next 10 years, and the owner
would retain the deed until the buyer finished
paying the installments.
After making the down payment, the buyer
moved onto the property and began clearing
some of the trees for a road and a cabin. He
regularly made the payments for several months
but then was killed by a falling tree. His properly
executed will conveyed his real estate to a friend
and the remainder of his estate, consisting of
personal property and $200,000 in cash, to his
son. During the next several months, his estate
failed to make payments on the installment
contract. The owner then notified the estate that
he was rescinding the deal and asserting ownership of the parcel, and offered to return the
amount the buyer had paid him, less expenses, as restitution.
The buyer’s estate initiated a quiet title action,
naming the owner, the friend, and the son as
parties. The estate’s filings indicated that it
was prepared to complete the conveyance and
redeem the land by paying the accelerated full
balance of the contract from the proceeds of the
estate. The friend believes he should receive title
to the parcel free of any obligation on the installment contract.
Which of the following doctrines is inapplicable?
(A) The doctrine of equitable conversion.
(B) The doctrine of part performance.
(C) The equitable mortgage doctrine.
(D) The doctrine of exoneration.
(C) The court will not apply the equitable mortgage doctrine in making its determination.
A landowner needing to raise money may “sell” the land to a person who will pay cash, giving the
“buyer” an absolute deed rather than a mortgage. If the court concludes that the deed was really given as security for a loan, rather than a true conveyance, it will treat the deed as an “equitable” mortgage and require that the lender foreclose it by judicial action, like any other mortgage.
Here, there was no such security arrangement between the owner and the buyer; rather, they entered into an installment land contract arrangement. Thus, the equitable mortgage doctrine would not support the court’s determination that the friend receive title to the parcel.
(A) is incorrect because equitable conversion is applicable here.
Under that doctrine, once a contract is made
and each party is entitled to specific performance, equity regards the purchaser as the owner of the real property. If the purchaser dies before title has passed, his interest is characterized as real property in his estate and will go to the taker of the estate’s real property on closing, which in this case is the friend.
(B) is incorrect. Although the Statute of Frauds applies to land sale contracts, the doctrine of part performance allows a court to order specific performance despite the absence of a writing if additional facts are present. In most jurisdictions, part performance can be established by two of the following:
(i) possession of the land by the purchaser;
(ii) making of substantial improvements; and/or (iii) payment of all or part of the purchase price.
Here, the buyer’s estate could be entitled to specific performance of the contract because he had (i) moved onto the property, (ii) begun clearing trees, and (iii) made several payments before his death.
(D) is incorrect. The court must apply the doctrine of exoneration, which is followed by a few states,
for the friend to receive the parcel free of the obligation on the installment contract.
The doctrine provides that when a testator makes a devise of real property that is subject to a mortgage, the devisee is entitled to have the mortgage satisfied out of the testator’s residuary estate.
Thus, if the court applies exoneration, it will order that the mortgage be paid from the $200,000 cash residuary that was left to the buyer’s son.
A seller entered into a written contract to
sell a tract of land to a buyer. The buyer was to
pay $1,500 per month for five years, at which
time the seller would deliver a warranty deed.
The contract was silent as to the quality of title
to be conveyed. After making 12 payments,
the buyer discovered that a neighbor had an
easement of way over the land, which was not
discussed at the time the seller and buyer entered into the contract. The neighbor had not used the easement over the previous year because she had been out of the country. On the basis of the easement, the buyer wishes to cancel the contract.
Which party is more likely to prevail?
(A) The seller, because the neighbor’s easement
has been extinguished.
(B) The seller, because the buyer has no basis
on which to rescind the contract.
(C) The buyer, because the obligation to convey
marketable title is implied.
(D) The buyer, because the seller has breached
the covenant against encumbrances.
(B) The seller is more likely to prevail because the buyer has no basis on which to rescind the
contract.
Absent a provision to the contrary, a contract for the sale of land contains an implied promise by the seller that she will deliver to the buyer a marketable title at the time of closing.
This promise imposes on the seller an obligation to deliver a title that is free from reasonable
doubt; i.e., free from questions that might present an unreasonable risk of litigation. Title is
marketable if a reasonably prudent buyer would accept it in the exercise of ordinary prudence.
An easement that reduces the value of the property (e.g., an easement of way for the benefit of a neighbor) generally renders title unmarketable.
If the buyer determines, prior to closing, that the seller’s title is unmarketable, he must notify the seller and allow a reasonable time to cure the defect.
If the seller is unable to acquire title before closing, so that title remains unmarketable, the buyer can rescind, sue for damages caused by the breach, or obtain specific performance with an abatement of the purchase price.
However, the buyer cannot rescind prior to closing on grounds that the seller’s title is unmarketable.
Where an installment land contract is used, the seller’s obligation is to furnish marketable title when delivery is to occur, e.g., when the buyer has made his final payment.
Thus, a buyer cannot withhold payments or seek other remedies on grounds that the seller’s title is unmarketable prior to the date of promised delivery.
Here, there is a valid easement on the property, but the seller has four years in which to cure this defect.Thus, the buyer cannot yet rescind on grounds that title is unmarketable.
A landowner devised her campground in her
will “to my niece, her heirs, and assigns, so
long as it is used for camping and recreational
purposes; if used for any other purpose during
her lifetime, then to the Girl Scouts of America.”
Subsequently, the landowner died. The residuary
clause of her will left all property not devised
in the remainder of the will to her daughter and
sole heir. Soon thereafter, the daughter died
intestate, her only heir being her son. Last month, the niece entered into a contract to sell the campground to a buyer for its reasonable
market value. After the buyer received the
title report called for in the contract, he refused
to proceed with the purchase, claiming that the
niece could not convey good title. The niece,
the Girl Scouts of America, and the buyer then
execute a new contract calling for the former
two parties to sell the property at the same
price to the latter. The jurisdiction follows the
common law Rule Against Perpetuities, and a
statute provides that future estates and interests
are alienable, and may be devised or inherited,
all in the same manner as possessory estates or
interests.
Should the buyer proceed with the new
purchase transaction?
(A) Yes, because good title can now be conveyed
by the sellers.
(B) Yes, if the Girl Scouts of America promises
never to use its right of entry should the
buyer use the property for other than
camping and recreational purposes.
(C) No, because the daughter’s son has not been
included as a party selling the property.
(D) No, because no one can convey good title to
the property during the niece’s lifetime.
(C) Good title cannot be obtained without the daughter’s son’s inclusion in the conveyance.
All contracts for the sale of land contain, unless the contract expressly provides otherwise, an implied covenant by the seller that she will deliver to the buyer a marketable title at the date of closing.
Private restrictions or encumbrances, including executory interests and possibilities of reverter,
will render title unmarketable unless the holders of those interests join in the transaction.
Here, the Girl Scouts have a valid executory interest that does not violate the Rule Against Perpetuities because their interest must vest, if at all, during the niece’s lifetime. However, the niece’s fee simple determinable is not limited in duration like the interest of the Girl Scouts. Thus, the niece’s heirs will only have a fee simple determinable rather than a fee simple absolute. Because the Girl Scouts’ interest vanishes on the niece’s death, and no other provision was made for the property if it should thereafter be used for noncamping or nonrecreational purposes, the transferor (the landowner) retained a possibility of reverter. This interest passed to the daughter and then to the daughter’s son, and so the son’s interest must be included in the conveyance.
(A) is wrong because the buyer has contracted to receive good title to a fee simple absolute without any restrictions on the use of the property. Unless the son joins in the contract to convey his possibility of reverter, good title cannot be conveyed.
(B) is wrong because the son’s interest is the one blocking good title; the Girl Scouts have already agreed to sell their interest. Furthermore, their interest is not a right of entry but a shifting executory interest that would automatically divest the buyer’s interest if he violated the use restriction, and so their promise would be meaningless.
(D) is wrong because the niece, the Girl Scouts, and the daughter’s son together can validly convey a fee simple absolute to the buyer.
Reformation is ….
… the remedy whereby the writing setting forth the agreement between the parties is changed to make it conform to the original intent of the parties.
Reformation may be available where there is a
mutual mistake (i.e., the writing does not conform to the original agreement and the parties are
not aware of the discrepancy).
As long as the parties were in agreement as to the terms before the contract was reduced to writing, reformation can be had regardless of whether both parties signed the contract without noticing the deviation from the oral agreement or one party knew of the deviation and the other did not.
Because land is considered unique,
specific performance is always appropriate for the enforcement of a valid land sale contract. This
option is available to
either the buyer or the seller.
When a contract for the sale of land is signed, equitable conversion takes place and it is, for all practical purposes, ….
the buyer’s land and the buyer’s risk.
A seller received a written offer in the mail
signed by a buyer to purchase the seller’s land for $50,000. The written offer was legally sufficient to form a written contract for the sale of the land.
The seller called the buyer and said that the offer
was acceptable, but that she wanted her attorney
to review it. The seller asked her attorney to
prepare a formal contract for the sale of the
land according to the same terms and conditions
in the written offer. When the attorney
had finished, the seller signed the contract and
mailed it to the buyer. Later that day, before the
buyer had received the contract, a developer
called the seller and offered to buy the land for
$60,000, which the seller accepted immediately
over the phone. The seller called the buyer and
told him that she had received and accepted a
higher offer. The seller then signed a written
contract to sell the land to the developer. When
the developer received the contract, he signed
it and then promptly and properly recorded it,
and sent the seller the specified down payment.
The buyer received the written contract from the
seller the next day. The recording statute in the
jurisdiction provides: “Any conveyance of an
interest in land, other than a lease for less than
one year, shall not be valid against a subsequent
purchaser for value, without notice thereof,
whose conveyance is first recorded.”
In an appropriate action brought by the buyer
against the seller and the developer for specific
performance and to quiet title, will the buyer
win?
(A) Yes, because the buyer’s written offer satisfies
the Statute of Frauds.
(B) Yes, because the contract of sale prepared
by the attorney satisfies the Statute of
Frauds.
(C) No, because the buyer never entered into a
binding contract with the seller.
(D) No, because the recording statute protects
the developer.
(D) The buyer will lose because the race-notice recording statute protects the developer.
The developer paid a fair price for the land and had no knowledge of the buyer’s claim to the land at the time he purchased the property. He would thus qualify as a bona fide purchaser for value and, because he was the first to record, he would have priority over the buyer.
(A) and (B) are incorrect because the recording act determines priority among purchasers of property while the Statute of Frauds deals only with the validity of an individual contract.
(C) is incorrect because _the seller’s
signing the contract, which contained the same terms and conditions as the buyer’s offer, *constituted an acceptance*, which became effective upon dispatch under the “mailbox rule” of contract law._